Question 1: Binomial distribution We are testing the landing performance of a new automated drone. The drone lands on the targeted area 80% of the time. We test the drone 12 times. Let X be the number of landings out of the targeted are.
a. Explain why the X is a binomial random variable and provide its characteristics.
b. What is the probability that the drone will land out of the targeted area exactly 4 times?
c. What is the probability that the drone will land out of the targeted area at most 4 times?
d. What is the expected value of X?
e. Explain the meaning of the expected value in the context of the story
f. What is the variance of X?
g. Given that drone missed the landing targeted area at most 4 times, what is the probability that it missed the target at most 2 times?
h. Given that drone missed the landing targeted area at most 4 times, what is the probability that it missed the target at least 2 times?
i. What is probability that X is within three standard deviations of the mean

Answers

Answer 1

a) The probability that X is within three standard deviations of the mean is approximately 1.

b) the probability that the drone will land out of the targeted area exactly 4 times is 0.00052.

c) The probability that the drone will land out of the targeted area at most 4 times is 0.1029

d) The expected value of X is 9.6.

e) The meaning of the expected value in the context of the story is average landing performance of the drone based on the given probability of success.

f) The variance of X is 0.7319.

g) The probability that it missed the target at most 2 times is 3.121.

h) The probability that it missed the target at least 2 times is 0.7319.

I) The probability that X is within three standard deviations of the mean is 1.3856.

The Binomial Distribution:

The binomial distribution is a discrete probability distribution that describes the number of successes in a fixed number of independent trials with a constant probability of success.

The characteristics of a binomial random variable include the number of trials (n), the probability of success (p), the number of successes (x), and the mean and variance of the distribution.

Here we have

Binomial distribution We are testing the landing performance of a new automated drone. The drone lands on the targeted area 80% of the time. We test the drone 12 times.

a. X is a binomial random variable because we have a fixed number of independent trials and each landing has only two possible outcomes (landing on the targeted area or landing outside of it) with a constant probability of success (0.8).

The characteristics of the binomial distribution are:

The number of trials is fixed (n=12)

Each trial has only two possible outcomes (success or failure)

The probability of success (p) is constant for each trial

The trials are independent of each other

b. P(X = 4) = (12 choose 4) × (0.8)⁴ × (0.2)⁸ = 0.00052

c. P(X< = 4) = P(X = 0) + P(X = 1) + P(X = 2) + P(X = 3) + P(X = 4)

= 0.0687 + 0.2060 + 0.3020 + 0.2670 + 0.1854 = 0.1029

d. E(X) = np = 120.8 = 9.6

e. The expected value of X represents the average number of successful landings (in the targeted area) we would expect to see in a sample of 12 landings.

In the context of the story, it tells us the average landing performance of the drone based on the given probability of success.

f. Var(X) = np(1-p) = 120.80.2 = 1.92

g. P(X<=2 | X<=4) = P(X<=2)/P(X<=4)

= (P(X=0) + P(X=1) + P(X=2))/(P(X=0) + P(X=1) + P(X=2) + P(X=3) + P(X=4))

= 0.3217/0.1029 = 3.121

h. P(X>=2 | X<=4) = 1 - P(X<2 | X<=4) = 1 - P(X<=1 | X<=4) = 1 - (P(X=0) + P(X=1))/(P(X=0) + P(X=1) + P(X=2) + P(X=3) + P(X=4)) = 1 - 0.2747/0.1029 = 0.7319

i. The standard deviation of a binomial distribution is √(np(1-p)). So, the standard deviation of X is √(120.80.2) = 1.3856. Three standard deviations above and below the mean would be 3*1.3856 = 4.1568.

Therefore,

The probability that X is within three standard deviations of the mean is approximately 1.

Learn more about Binomial Distribution at

https://brainly.com/question/31197941

#SPJ4


Related Questions

The function y=f(x) is graphed below. What is the average rate of change of the function f(x) on the interval 4≤x≤7?

Answers

Answer:

5

Step-by-step explanation:

The average rate of change of function f(x) from x = a to x = b is

[tex] \dfrac{f(b) - f(a)}{b - a} [/tex]

The graph shows f(7) = 10, and f(4) = -5.

[tex] \dfrac{f(7) - f(4)}{7 - 4} = [/tex]

[tex] = \dfrac{10 - (-5)}{7 - 4} [/tex]

[tex] = \dfrac{15}{3} [/tex]

[tex]= 5[/tex]

!!will give brainliest!!!

Find WZ to the nearest tenth.
Assume that segments that appear
to be tangent are tangent.

Answers

The measure of secant WZ = 5 units

We know that the Secant-Tangent theorem states that, 'when a secant and tangent of a circle intersect at the same external point, then the product of the measure of the secant segment and its external part equals the square of the measure of the tangent segment.'

Here, VW is a tanget to a circle at point V and ZW is a secant of a circle.

From  Secant-Tangent theorem,

ZY × YW = VW²

(x + 3) × (x) = (x + 1)²

We solve this equation for x.

x² + 3x = x² + 2x + 1

3x - 2x = 1

x = 1

So, the length of WY = 1 unit

So, the length of ZY would be,

x + 3

= 1 + 3

= 4

and the length of WZ = WY + YZ

                                    = 1 + 4

                                    = 5 units

This is the required length of WZ  

Learn more about the secant here:

https://brainly.com/question/23026602

#SPJ1

Braxton was holding a bake sale to raise money for his field trip. He sold cookies for $2 each, muffins for $3 each, and lemonade for $2 a bottle. If he sold 15 cookies, 10 muffins, and 26 bottles of lemonade, how much money did he raise for his field trip?


$126

$112

$134

Answers

Answer:

$2(15) + $3(10) + $2(26) = $30 + $30 + $52

= $112

(2x15)+(3x10)+(2x26)
=112

Therefore he raised $112 for his field trip.

In the triangle shown below, find the value of a.

Answers

Answer:

the value is 55

Step-by-step explanation:

gouge-em cable company is the only cable television service company licensed to operate in backwater county. most of its costs are access fees and maintenance expenses. these fixed costs total $640,000 monthly. the marginal cost of adding another subscriber to its system is constant at $2 per month. gouge-em's demand curve can be determined from the data in the accompanying table. Complete the following table by computing the total revenue, total cost, and profit at each of the various subscription prices. Gouge-em will charge ____________ for its cable services, earning them a profit of $____________ thousand. Now suppose the Backwater County Public Utility Commission has the data and believes that cable subscription rates in the county are too expensive and that Gouge-em's profits are unfairly high What regulated price will it set so that Gouge-em makes only a normal rate of return on its investment? A. $5 B. $10 C. $15 D. $20

Answers

Gouge-em Cable Company will charge $30 for its cable services, earning them a profit of $70 thousand. The Backwater County Public Utility Commission will set the regulated price at $15 so that Gouge-em makes only a normal rate of return on its investment.

To find the optimal price that Gouge-em Cable Company should charge for its cable services, we need to calculate the total revenue, total cost, and profit at each of the various subscription prices. The demand curve provided gives us the number of subscribers that will sign up at different prices.

Price Quantity Demanded Total Revenue Total Cost Profit

$10 100 $1,000 $640,200 -$639,200

$20 80 $1,600 $640,160 $959,840

$30 60 $1,800 $640,120 $1,159,880

$40 40 $1,600 $640,080 $959,920

$50 20 $1,000 $640,040 $359,960

To maximize profit, Gouge-em Cable Company should charge the price where marginal revenue equals marginal cost. Since the marginal cost of adding another subscriber is constant at $2 per month, we can calculate marginal revenue by taking the difference in total revenue between two adjacent price points. For example, the marginal revenue of charging $20 instead of $10 is $600 ($1,600 - $1,000) for 20 additional subscribers.

The table shows that the optimal price is $30, where marginal revenue equals marginal cost at $2 per subscriber, and profit is maximized at $1,159,880.

However, the Backwater County Public Utility Commission believes that Gouge-em's profits are unfairly high, so it wants to regulate the price to ensure a normal rate of return on investment. A normal rate of return is typically around 10% of total investment. Gouge-em's total investment is the sum of fixed costs divided by the monthly profit margin:

Total investment = Fixed costs / Monthly profit margin

= $640,000 / ($1,159,880 / 5)

= $27,627,724.51

A 10% return on investment is $2,762,772.45 per year, or $230,231.04 per month. To earn this amount, Gouge-em needs to charge a price that covers its total costs plus the normal rate of return, which is:

Regulated price = Total cost / Quantity demanded + Normal rate of return / Quantity demanded

= $640,000 / 60 + $230,231.04 / 60

= $10.17

Therefore, the Backwater County Public Utility Commission will set the regulated price at $15, which is a round number close to the calculated price of $10.17. At this price, Gouge-em will make a normal rate of return on its investment.

To learn more about Public Utility here:

brainly.com/question/30748959#

#SPJ11

NEED HELP ASAP.
ΔABC has vertices at (-4, 4), (0,0) and (-5,-2). Find the coordinates of points A, B and C after a reflection across y= x.

Point A': ___________

Point B': ___________

Point C': ___________

Answers

The reflected coordinates of the vertices A, B, and C are:

A' = (4, -4)
B' = (0, 0)
C' = (-2, -5)

To reflect a point across the line y = x, we swap its x and y coordinates. So to find the reflected coordinates of each vertex, we just need to swap their x and y values.

Let's start with vertex A(-4, 4):

After reflecting across y = x, its coordinates become (4, -4).

Now, let's move to vertex B(0,0):

After reflecting across y = x, its coordinates remain the same, because any point on the line y = x is its own reflection.

Finally, we have vertex C(-5, -2):

After reflecting across y = x, its coordinates become (-2, -5).

Therefore, the reflected coordinates of the vertices A, B, and C are:

A' = (4, -4)

B' = (0, 0)

C' = (-2, -5)

Learn more about transformation here:

https://brainly.com/question/18065245
#SPJ1

SSS: Cut three pieces of string. Make each piece of string the length of one of the sides of the original triangle. Put the string together to form a triangle and trace the triangle on a separate piece of paper. Measure the angles of the triangle with your protractor. Answer the following questions in your math journal: Are the lengths of the sides and the measures of the angles of the triangle you created the same as the original triangle? Rearrange the string to make a different triangle. Is there any way to create a triangle that has different angle measures? SAS: Choose two sides of the original triangle. Cut two pieces of string and make each piece of string the length of one of those sides. Measure out the angles at both endpoints of the side that you chose. Draw the angles with the given measurements. Put the string together to form the sides of that angle and trace them. Draw in the third side of the triangle. Measure the third side that you drew and the two angles adjacent to that side. Answer the following questions in your math journal: Are the lengths of the sides and the measures of the angles of the triangle you created the same as the original triangle? Draw the starting angle elsewhere on your paper and rearrange the string to make a different triangle. Is there any way to create a triangle whose third side has a different length? ASA: Choose one side of the original triangle. Cut one piece of string and make the piece of string the length of that side. Trace the string on a separate sheet of paper. Measure out the angles at both endpoints of the side that you chose. Draw the angles with the given measurements. Extend the sides of the angles until they intersect and form a triangle. Measure the two sides that you drew and the angle between them. Answer the following questions in your math journal: Are the lengths of the sides and the measures of the angles of the triangle you created the same as the original triangle? Rearrange the string and re-draw the two starting angles to make a different triangle. Is there any way to create a triangle that has different side lengths?​

Answers

SSS, SAS, and ASA are three distinct techniques for figuring out if a triangle is validly formed by three provided side lengths, two sides and an included angle, or two angles and an included side, respectively.

In the SSS technique, three pieces of string are organised into a triangle by first being cut to the lengths of its sides. In the SAS approach, a triangle is made using two sides and an added angle. The angles are measured and drawn on a separate piece of paper, and the two sides are symbolised by two strands of thread.

In the ASA technique, a triangle is made up of one side and two neighbouring angles. On a different piece of paper, a piece of thread is traced to the length of the side. The two sides are stretched until they connect to create a triangle by measuring and drawing the two neighbouring angles. The string pieces will form a triangle if their side lengths and angle measurements match those of the original triangle.

The triangle inequality theorem, which asserts that the total of any two sides of a triangle must be greater than the third side, is not satisfied by the string pieces in any of the three approaches.

Learn more about SSS visit: brainly.com/question/3999145

#SPJ4

Find the slope and
-intercept from the following graph of a linear equation.

Answers

Answer:

Slope = 4

y-intercept = (0, 3)

Step-by-step explanation:

The slope of a line is a measure of its steepness. It represents how much the line rises or falls as it moves horizontally.

The slope of a line is calculated by dividing the change in y by the change in x between any two points on the line: "rise over run".

From inspection of the given graph, the y-value increases by 4 units each time the x-value increases by 1 unit, . Therefore, the rise is 4 units and the run is 1 unit. As 4/1 = 4, then the slope of the line is 4.

The y-intercept is the point at which the line intersects the y-axis, so when x = 0.

From inspection of the given graph, the line crosses the y-axis at 3, the y-intercept of the line is (0, 3).

What is the length of PQ¯¯¯¯¯?

Enter your answer as a decimal in the box. Round only your final answer to the nearest tenth.


km

A horizontally-aligned triangle P Q R. Side P R is labeled as 6 kilometers. Side R Q is labeled as 9 kilometers. Angle R is labeled as 34 degrees.

Answers

The length of PQ is given as follows:

PQ = 5.24 km.

What is the law of cosines?

The Law of Cosines is a trigonometric formula that relates the lengths of the sides of a triangle to the cosine of one of its angles. It is also known as the Cosine Rule.

The Law of Cosines states that for any triangle with sides a, b, and c and angle C opposite to side c, the following equation holds true:

c^2 = a^2 + b^2 - 2ab cos(C)

For the angle of 34º, we have that:

PQ is the opposite segment.6 km and 9 km are the adjacent segments.

Hence the length of PQ is obtained as follows:

(PQ)² = 6² + 9² - 2 x 6 x 9 x cosine of 34 degrees

PQ = sqrt(6² + 9² - 2 x 6 x 9 x cosine of 34 degrees)

PQ = 5.24 km.

More can be learned about the law of cosines at https://brainly.com/question/4372174

#SPJ1

12 1 point Suppose P(A) = 0.8, P(B) = 0.5 and P(AUB) = 0.9. Which one of the following statements is true? Events A and B are independent. - Events A and B are both mutually exclusive and independent. The probability of the intersection of A and B is 0.1. Events A and B are mutually exclusive.

Answers

Only statement left is "Events A and B are mutually exclusive," which is also not true since P(A∩B) = 0.1, which is greater than 0.

Thus, none of the statements is true.

None of the statements is true.

If events A and B were independent, then P(A∩B) = P(A)P(B) = 0.4, which is not equal to 0.1.

If events A and B were mutually exclusive, then P(A∩B) = 0, which is not equal to 0.1.

Therefore, neither of the first two statements is true.

Since P(A∪B) = P(A) + P(B) - P(A∩B), we have P(A∩B) = 0.4, which is not equal to 0.1. Therefore, the third statement is not true.

The only statement left is "Events A and B are mutually exclusive," which is also not true since P(A∩B) = 0.1, which is greater than 0.

Thus, none of the statements is true.

To learn more about statements visit:

https://brainly.com/question/30238773

#SPJ11

From the observation deck of a skyscraper, Lavaughn measures a 42° angle of
depression to a ship in the harbor below. If the observation deck is 872 feet high,
what is the horizontal distance from the base of the skyscraper out to the ship?
Round your answer to the nearest hundredth of a foot if necessary.

Answers

Answer:

  968.45 ft

Step-by-step explanation:

You want the horizontal distance to a ship if the angle of depression to it is 42° from a station 872 feet high.

Tangent

The tangent relation is ...

  Tan = Opposite/Adjacent

In the model of this problem, the distance adjacent to the angle of depression is the distance to the ship (x). The opposite distance is the height of the observation point, and the angle is the angle of depression:

  tan(42°) = (872 ft)/x

  x = (872 ft)/tan(42°) ≈ 968.45 ft

The horizontal distance to the ship is 968.45 feet.

<95141404393>

Which graph represents the solution to this system of equations?
2x+2y=6
2x+4y=12

Answers

The solution to the system of equations is x = 0 and y = 3, or the ordered pair (0, 3).

To solve this system of equations, we can use the method of elimination. We want to eliminate one of the variables so that we can solve for the other. In this case, we can eliminate x by subtracting the first equation from the second equation, since the coefficients of x are the same and will cancel out:

(2x + 4y) - (2x + 2y) = 12 - 6

Simplifying the left side and right side of the equation, we get:

2y = 6

y = 3

Now that we have solved for y, we can substitute this value back into either equation to solve for x. Using the first equation, we get:

2x + 2(3) = 6

x = 0

Therefore, the solution to the system of equations is x = 0 and y = 3, or the ordered pair (0, 3).

Learn more about the solution to the equation here:
https://brainly.com/question/545403

#SPJ1

Find the perimeter of △VWU. Round your answer to the nearest tenth

Answers

The perimeter of the shape based on the information will be 109.8.

How to calculate the perimeter

The smaller triangle contains the length of the side facing 34 degrees is 27. The scale factor for separating the smaller from the larger is 27/30 = 9/10 or 0.9.

Similarly, the side facing 51 degrees in the larger is 40, whereas it is 36 in the smaller.

Hence, the ratio remains 36/40 = 9/10 or 0.9.

In essence, the smaller triangle will have 0.9 times the circumference of the larger triangle.

The larger's perimeter is simply the sum of the side lengths.

This is what we have:

(52 + 30 + 40) = 122

As in the case of the smaller; 122 * 0.9 = 109.8

Learn more about perimeter on

brainly.com/question/24180015

#SPJ4

Suppose the following list of events describes all of the economic activity resulting from an increase in government spending Suppose that at each step after the initial one, the marginal propensity to consume is 0.58 and the tax rate is 12% Step 0. The government spends $5500 on meat to host a very large dinner for foreign diplomats Step A. The butcher takes the income earned by selling the meat saves some and spends the rest on a wedding cake for his daughter. Step B. The baker who produced the wedding cake saves some of her earnings and uses the rest to purchase beautiful candlesticks as gifts for all of her friends. Step C. The local candlestick maker saves some of his revenue for retirement and spends the rest on building materials to improve his house. Instructions: Modify the settings in the interactive tool to represent this event. Then click 'Spending Rounds and use the table to answer the following questions. Round answers to the nearest cent, if necessary How much does the candlestick maker earn for selling the candlesticks? SDS How much does the candlestick maker spend on building materials?

Answers

To find out how much the candlestick maker earns for selling the candlesticks and how much he spends on building materials, we need to follow the marginal propensity to consume (MPC) and tax rate through each step.

Step 0: Government spends $5,500 on meat for foreign diplomats.
Step A: Butcher's income is $5,500. He pays 12% in taxes, so his after-tax income is $5,500 * (1 - 0.12) = $4,840. He spends 0.58 * $4,840 = $2,806.80 on a wedding cake.
Step B: Baker's income is $2,806.80. She pays 12% in taxes, so her after-tax income is $2,806.80 * (1 - 0.12) = $2,470.99. She spends 0.58 * $2,470.99 = $1,433.17 on candlesticks.
Step C: Candlestick maker's income is $1,433.17. He pays 12% in taxes, so his after-tax income is $1,433.17 * (1 - 0.12) = $1,261.19.

So, the candlestick maker earns $1,433.17 for selling the candlesticks.

Now, we calculate how much the candlestick maker spends on building materials:

Candlestick maker spends 0.58 * $1,261.19 = $731.09 on building materials.

Your answer: The candlestick maker earns $1,433.17 for selling the candlesticks and spends $731.09 on building materials.

To learn more about Selling - brainly.com/question/30615010

#SPJ11

Work out the value of
[tex]( \frac{8}{27} ) {}^{ \frac{ 4}{3} } [/tex]

Answers

The expression is simplified to 16/81

What are index forms?

Index forms are simply described as those mathematical forms that are used in the representation of numbers or variables in more convenient forms.

Index forms are also referred to as;

Scientific notationStandard forms

From the information given, we have;

(8/27)⁴/³

To determine the value

Find the cube root, we get;

(∛8/27)⁴

(2/3)⁴

Find the value of the exponents

16/81

Thus, the value is 16/81

Learn more about index forms at: https://brainly.com/question/15361818

#SPJ1

13. Solve the following system of linear equations by substitution, elimination or by vraphing: y = 3x - 1 8x - 2y = 14

Answers

To solve the system of linear equations:

y = 3x - 1
8x - 2y = 14

We can use either the substitution or elimination method.

Substitution method:
Step 1: Solve one of the equations for one variable (in this case, y).
y = 3x - 1
Step 2: Substitute the expression for y into the other equation.
8x - 2y = 14
8x - 2(3x - 1) = 14
Step 3: Simplify and solve for the remaining variable (in this case, x).
8x - 6x + 2 = 14
2x = 12
x = 6
Step 4: Substitute the value of x back into one of the original equations and solve for the other variable (in this case, y).
y = 3x - 1
y = 3(6) - 1
y = 17
Therefore, the solution to the system of linear equations is (6, 17).

Elimination method:
Step 1: Multiply one or both equations by a constant so that the coefficients of one variable are additive inverses (in this case, the coefficients of y).
y = 3x - 1
8x - 2y = 14
Multiplying the first equation by 2, we get:
2y = 6x - 2
Multiplying the second equation by -1, we get:
-8x + 2y = -14
Step 2: Add the two equations to eliminate y.
-8x + 2y = -14
+ 2y = 6x - 2
-8x + 0 = 4x - 16
12x = 16
x = 4/3
Step 3: Substitute the value of x back into one of the original equations and solve for the other variable (in this case, y).
y = 3x - 1
y = 3(4/3) - 1
y = 1
Therefore, the solution to the system of linear equations is (4/3, 1).

Graphing method:
Step 1: Graph each equation on the same coordinate system.
y = 3x - 1 is a line with slope 3 and y-intercept -1.
8x - 2y = 14 can be rewritten as y = 4x - 7, which is also a line with slope 4 and y-intercept -7.
Step 2: Determine the point of intersection of the two lines, which is the solution to the system of equations.
The two lines intersect at (6, 17).
Therefore, the solution to the system of linear equations is (6, 17).

Learn more about linear equations: https://brainly.com/question/2030026

#SPJ11

What is the product of 1. 0 * 10^3 and 2. 0 x 10^5 expressed in scientific notation. HEEEELLLLPPPPP PLEASEEEEEEEE

Answers

The product of the exponents is given by A = 2.0 x 10⁸

Given data ,

Let the first number be p = 1 x 10³

Let the second number be q = 2 x 10⁵

From the laws of exponents , we get

mᵃ×mᵇ = mᵃ⁺ᵇ

A = p x q

On simplifying , we get

A = 1 x 10³ x 2 x 10⁵

A = 2 x 10³⁺⁵

A = 2.0 x 10⁸

Hence , the equation is A = 2.0 x 10⁸

To learn more about exponents click :

https://brainly.com/question/28966438

#SPJ1

The sum of half a number, n, and 15 is 24. What is the value of the number n?

Answers

Answer:

n = 18

Step-by-step explanation:

We can use the following equation to solve for n:

[tex]1/2n+15=24\\1/2n=9\\n=18[/tex]

If we check, we see that half of 18 is 9 and 9 + 15 is indeed 24

Solve (4-2x) (3+5x) using the foil method

Answers

12 + 14x - 10x^2

.............................

Refer to the attached image.

Show all your calculations in order to get a full credit. 17.17 Given these data X 5 10 15 20 25 30 35 40 45 50 у 17 24 31 33 37 37 40 40 42 41 use least-squares regression to fit (a) a straight line, y = a0 + a1x (b) a power equation, y = axb (c) a saturation-growth-rate equation y = a* and (d) BONUS:a parabola y = a0+ a1x + a2x2 (e) In each case, Program in Matlab and check results done in Parts a, b, and c. Plot the data and the equation. For each case find Coefficient of determination and Correlation coefficient Is any one of the curves -superior? If so, justify.

Answers

Coefficient of determination and Correlation coefficient Is any one of the curves -superior is Y = [2.83321 3.17805 3.43399 3.49651 3.61092 3.61092 3.68888 3.68888 3.73767 3.71357]

n = 10

(a) Fitting a straight line using least-squares regression:

To find the equation of the line of best fit, we need to calculate the slope and intercept using the following formulas:

a1 = (nΣ(xy) - ΣxΣy) / (nΣx^2 - (Σx)^2)

a0 = y - a1x

where n is the sample size, Σ denotes the sum of, x and y are the mean of X and Y respectively.

Substituting the given values, we get:

n = 10

Σx = 275

Σy = 342

Σxy = 11745

Σx^2 = 8250

x = 27.5

y = 34.2

a1 = (1011745 - 275342) / (108250 - 275^2) = 0.8929

a0 = 34.2 - 0.892927.5 = 10.3143

Therefore, the equation of the line of best fit is:

y = 10.3143 + 0.8929x

To check these results using Matlab, we can use the following code:

x = [5 10 15 20 25 30 35 40 45 50];

y = [17 24 31 33 37 37 40 40 42 41];

mdl = fitlm(x,y)

The output should show the intercept and slope values, which match our calculated values. We can also plot the data and the line of best fit using the following code:

plot(x,y,'o')

hold on

xfit = 5:50;

yfit = 10.3143 + 0.8929*xfit;

plot(xfit,yfit,'-')

(b) Fitting a power equation using least-squares regression:

A power equation has the form y = ax^b, where a and b are constants. To fit a power equation using least-squares regression, we need to transform the equation into a linear form by taking the logarithm of both sides:

log(y) = log(a) + b*log(x)

Let Y = log(y) and X = log(x), then the equation becomes:

Y = log(a) + bX

This is now in the form of a straight line, y = a0 + a1x, where a0 = log(a) and a1 = b. We can use the same formulas as in part (a) to find the slope and intercept of the line of best fit:

a1 = (nΣ(XY) - ΣXΣY) / (nΣX^2 - (ΣX)^2)

a0 = Y - a1x

where X and Y are the means of X and Y respectively.

Substituting the given values, we get:

X = [0.69897 1 1.17609 1.30103 1.39794 1.47712 1.54407 1.60206 1.65321 1.69897]

Y = [2.83321 3.17805 3.43399 3.49651 3.61092 3.61092 3.68888 3.68888 3.73767 3.71357]

n = 10

ΣX = 12.05009

ΣY =

To learn more about formulas visit:

https://brainly.com/question/28647690

#SPJ11

Please help 5 points Question in picture

Identify the type of slope each graph represents

A) Positive
B) Negative
C) Zero
D) Undefined

Answers

When reading a graph it’s the same as reading most books from left to right and since the line goes up from left to right it is a positive slope.

Positive the slope is going up therefore is positive

On average, Logan drinks 2/3 of a 6-ounce glass of water in 2 1/4 hours. How much water does he drink, in glasses per hour?

Answers

Logan drinks 8/3 glasses of water per hour on average.

To find how much water Logan drinks in glasses per hour, we need to divide the amount of water he drinks by the time it takes him to drink it.

First, let's convert 2/3 of a 6-ounce glass of water into ounces:

2/3 x 6 = 4 ounces

So Logan drinks 4 ounces of water in 2 1/4 hours. To convert 2 1/4 hours to a mixed number of hours, we need to express it with the same denominator as the fraction:

2 1/4 = 9/4

Now we can divide the amount of water (4 ounces) by the time (9/4 hours):

4 ÷ (9/4) = 16/9

So Logan drinks 16/9 ounces of water per hour. To express this in glasses per hour, we need to divide by the size of one glass:

6 ounces/glass

(16/9 ounces/hour) / (6 ounces/glass) = 8/3 glasses per hour.

Learn more about  per hour

https://brainly.com/question/3818718

#SPJ4

Greenfields is a family operated business that manufactures fertilisers. One of its products is a liquid plant feed into which certain additives are put to improve effectiveness. Every 10,000 litres of this feed must contain at least 480 g of addir tive A, 800 g of additive B and 640 g of additive C. Greenfields can purchase two ingredients X and Y) that contain these three additives. This information, together with the cost of each ingredient, is given below as follows: Ingredient Ingredient Y Additive A Additive B Additive C Cost per litre 29 89 59 109 10g 49 £50 $25 Both ingredients require specialist storage facilities and as such no more than 120 litres of each can be held in stock at any one time. Greenfields' objective is to determine how many litres of each ingredient should be added to every 10,000 litres of plant feed so as to minimise costs.

Answers

Minimise cost is given by  50X + 25Y

To determine how many litres of each ingredient (X and Y) should be added to every 10,000 litres of plant feed to minimise costs while meeting the additive requirements, follow these steps:

1. Set up the constraints based on additive requirements:
- 10A_X + 29A_Y ≥ 480 (Additive A)
- 49B_X + 89B_Y ≥ 800 (Additive B)
- 59C_X + 109C_Y ≥ 640 (Additive C)

2. Set up the constraints for the storage limitations:
- X ≤ 120 (Ingredient X storage)
- Y ≤ 120 (Ingredient Y storage)

3. Define the objective function to minimise cost:
- Cost = 50X + 25Y

4. Use linear programming techniques to solve the system of inequalities and find the optimal values of X and Y that minimise the cost function while satisfying all the constraints.

5. The optimal solution for X and Y will indicate the number of litres of each ingredient that should be added to every 10,000 litres of plant feed to minimise costs while meeting the additive requirements and storage limitations.

To learn more about Linear programming

https://brainly.com/question/29421622

#SPJ11

which of the following is not a characteristic for a normal distribution? group of answer choices it is symmetrical the mean is always zero it is symmetric about its mean it is a bell-shaped distribution

Answers

The characteristic that is not true for a normal distribution is "the mean is always zero".

While it is true that the normal distribution is symmetrical, symmetric about its mean, and has a bell-shaped distribution, the mean of a normal distribution can be any number, not just zero. The mean of a normal distribution represents the center of the distribution and can be positive, negative, or zero, depending on the data being analyzed. It is important to note that a normal distribution is a statistical concept that is used to describe the distribution of a set of data, and it is often used in various fields such as finance, engineering, and science. The normal distribution is known for its properties such as the central limit theorem, which states that the sum of a large number of independent random variables will be approximately normally distributed. In conclusion, the normal distribution is a symmetrical, bell-shaped distribution that is centered around its mean, but the mean can be any number, not just zero.

Know more about normal distribution here:

https://brainly.com/question/29509087

#SPJ11

18. Does the rule y = 6x² represent an exponential function?
Oyes
Ono

Answers

Answer: No it does not represent an exponential function.

Step-by-step explanation:

Hope it helps!

Good luck!!!

Determine how many terms of the following convergent series must be summed to be sure that the remainder is less than 10−2
[infinity]∑k=1(−1)k+1k4

Answers

There are 16 terms of the convergent series must be summed to be sure that the remainder is less than 10⁻²[infinity]∑k=1(−1)k+1k4

The alternating series estimation theorem can be used to determine an upper bound for the error in approximating the total of the series by summing a finite number of terms. As an example of an alternating sequence of the form:

∑(-1)^(n-1) b_n

The inaccuracy in approximating the series total by adding the first n terms equals the absolute value of the (n+1)th term:

|(-1)^n b_n+1|

In this case, we have:

∑k=1^∞ (-1)^(k+1) k^4

So the (n+1)th term is:

(-1)^n+1 (n+1)^4

To verify that the residual is smaller than 10(-2), we must find the smallest n such that:

|(-1)^n+1 (n+1)^4| < 10^(-2)

So let us try n = 1:

|(-1)^2 (2)^4| = 16 > 10^(-2)

So let us try n = 2:

|(-1)^3 (3)^4| = 81 > 10^(-2)

This approach can be repeated until we find the smallest value of n that meets the inequality. However, because this is time-consuming, we can use a calculator to compute the terms and check the inequality. As a result, we discover that n = 6 is the least value that works:

|(-1)^7 (7)^4| = 2401 > 10^(-2)

As a result, we must add the first sixteen terms of the convergent series to ensure that the remainder is less than 10(-2).

To learn more about convergent visit:

https://brainly.com/question/15415793

#SPJ11

Which choice is the slope intercept equation of the line shown below

Answers

Answer:

-2,2 + (2)-4 =?

Step-by-step explanation:

if you add y+-3r that would actually be the correct answer

Answer:

Choice C  y = -3x - 4

Step-by-step explanation:

slope is negative (line slants down), so you can toss out answer D.

y-intercept is -4 (where the line crosses the y axis), so you can toss out answers  A and B.

That leaves C as the right answer.

Just to prove that the slope = -3, calculate it:

y = (-4-2) / (0--2) = -6/2 = -3

If a cell group is formatted with multiple conditional formats, the rules are applied _______.
a. based on the hierarchy of the rule type
b. in the order in which they are created
c. based on which rule best applies to the first cell in the range
d. in alphanumeric order by the name of the rule

Answers

If a cell group is formatted with multiple conditional formats, the rules are applied in the order in which they are created.

It is needed to find the order that the rules are applied when a cell group is formatted with multiple conditional formats.

For a cell group, when multiple conditional formats are used, then the last rule that is added is the one that will be done first

However, this can be changed by clicking on the conditional formatting and then manage rules.

So the order of the rules will be of the order that the rules are created.

Hence the correct option is b.

Learn more about Conditional Formatting here :

https://brainly.com/question/30166920

#SPJ12

Evaluate each problem:
Tan 5pi/4

Sin 3pi/2

Cos 7pi/4

Answers

The values of each of the given trigonometric ratios are:

Tan 5pi/4 = 1

Sin 3pi/2 = -1

Cos 7pi/4 = 1/√2

How to solve trigonometric problems in radians?

There are three main trigonometric ratios and they are:

sin x = opposite/hypotenuse

cos x = adjacent/hypotenuse

tan x = opposite/adjacent

1) tan 5pi/4 when converted to degrees is tan 225 and using a calculator equals 1

2) Sin 3pi/2 when converted to degrees is sin 270 and using a calculator equals -1

3) Cos 7pi/4 when converted to degrees is cos 315 and using a calculator equals 1/√2

Read more about Trigonometry in radians at: https://brainly.com/question/19278379

#SPJ1

Do 4 in, 2 in, 8 in make a triangle and what kind ?

Answers

No, 4 in, 2 in, and 8 in do not make a triangle.

We have,

To determine whether 4 in, 2 in and 8 in make a triangle, we need to check if the sum of the two smaller sides is greater than the longest side.

If this condition is satisfied, then the three sides can form a triangle.

In this case, the two smaller sides are 2 in and 4 in, and the longest side is 8 in.

Therefore, we need to check if:

2 in + 4 in > 8 in

This simplifies to:

6 in > 8 in

Since this statement is not true, we can conclude that 4 in, 2 in, and 8 in cannot form a triangle.

Thus,

No, 4 in, 2 in, and 8 in do not make a triangle.

Learn more about triangles here:

https://brainly.com/question/25950519

#SPJ1

Other Questions
Which medication should be used with caution in a patient with a sulfa allergy? Benicar Cardura Edarbyclor Micardis Word combinations such as basic fundamentals, anonymous stranger, and future plans are examples of A) predicates.B) redundancies.C) split modifiers.D) camouflaged verbs.E) parallel terms. Which of these is the controlling idea of the section titled The Understory and Floor? Responses A The understory and floor of the tropical rainforest cannot support an abundance of life due to its hot and wet climate.The understory and floor of the tropical rainforest cannot support an abundance of life due to its hot and wet climate. B Plants and animals learn to live in a tropical ecosystem because they are not adapted to live outside these layers.Plants and animals learn to live in a tropical ecosystem because they are not adapted to live outside these layers. C Life at the understory and floor have specialized adaptations and functions in the tropical rainforest ecosystem.Life at the understory and floor have specialized adaptations and functions in the tropical rainforest ecosystem. D Plants growing on the floor of the tropical rainforest are distinct from the plants inhabiting the canopy.Plants growing on the floor of the tropical rainforest are distinct from the plants inhabiting the canopy. A coupon book has a coupon for 10% off a ticket at a theme park. A ticket usually costs $80. How much would a visitor save with the coupon? When you're composing a persuasive message, all of the following except ________ are common mistakes that you need to avoid.A) using a hard-sell approachB) promoting compromiseC) relying solely on great argumentsD) assuming that persuasion is a one-shot effortE) using a well-crafted story xyz stock price and dividend history are as follows: year beginning-of-year price dividend paid at year-end 2018 $ 124 $ 4 2019 135 4 2020 115 4 2021 120 4 an investor buys six shares of xyz at the beginning of 2018, buys another two shares at the beginning of 2019, sells one share at the beginning of 2020, and sells all seven remaining shares at the beginning of 2021. what is the dollar-weighted rates of return or irr for the investor? (do not round intermediate calculations. round your answer to x.xx% format.) The energy of light is called electromagnetic radiation. In the electromagnetic spectrum, photosynthesis makes use of which specific wavelengths?A) the entire electromagnetic spectrumB) X-raysC) ultraviolet radiationD) visible lightE) infrared radiation In a recent year (365 days), a hospital had 5742 births.a. Find the mean number of births per day.b. Find the probability that in a single day, there are 18 births.c. Find the probability that in a single day, there are no births. Would 0 births in a single day be a significantly low number of births?a. The mean number of births per day is 15.7.(Round to one decimal place as needed.)b. The probability that, in a day, there are 18 births is 0.07970.(Do not round until the final answer. Then round to four decimal places as needed.)c. The probability that, in a day, there are no births is(Round to four decimal places as needed.) A different sample box has a height that is twice as long as the original box described in Problem 1. What is thevolume of this sample box? How does the volume of this sample box compare to the volume of the sample box insProblem 1 Which statements are always true regarding the diagram? Select three options loss/change of job, death of a loved one, change in financial status, divorce, abortion, or severe mental/physical illness are all examples of what kind of crisis? FACTORING PUZZLEUse the digits 0-9 to fill in the squares. Each digit can be used only once.x - x -x - 1X +KeiraX1 = (x + 2)(x-x +-= (x-2)(x -x + 1= (x +x - 24 = (x-6)(x +=(x + 2)Source: Public Schools of North Carolina Resources for Algebra Four Alkaline D batteries are placed in series. What is the total voltage of the batteries? A. 1.5 VB. 6 VC. 9 VD. 36 V While caring for an older adult client, which symptom would require an immediate reassessment of the client's needs and plan of care?A. Memory loss or confusionB. Neglect of self-careC. Increased daily fatigueD. Withdrawal from usual activities The base of the pyramid is a rhombus with a side of 4.5 cm, and the largest diagonal is 5.4 cm. Calculate the area and volume of the pyramid if each side wall makes an angle of 45 with the plane of the base Which of the following is not one of the six main elements in the business intelligence environment discussed in this chapter?A) Managerial users and methodsB) Organizational environmentC) User interfaceD) Data from the business environmentE) Delivery platform what is a feature of the ductus deferens that enables it to propel sperm toward the ejaculatory ducts? the smooth muscular layers enable the ductus (vas) deferens to transport sperm from the epididymis to the ejaculatory ducts via peristalsis. the skeletal muscular layers enable the ductus (vas) deferens to transport sperm from the epididymis to the ejaculatory ducts via peristalsis. the smooth muscular layers enable the ductus (vas) deferens to transport sperm from the ejaculatory ducts to the epididymis via peristalsis. the skeletal muscular layers enable the ductus (vas) deferens to transport sperm from the ejaculatory ducts to the epididymis via peristalsis. The ls command displays the contents of a directory and includes several options.From the list on the left, drag an option to its correct description on the right.-a displays all directory contents, including hidden content.-l displays extended information, including the owner, modified date, size, and permissions.-R displays the contents of a directory and all of its subdirectories.-d displays directories but not files.-r reverses the sort order. what is social insurance? group of answer choices programs that people are required to contribute to that are designed to provide economic assistance to individuals in times of need programs that individuals may opt to invest in so that they have more income should they ever need it during future crises programs that the government runs so that investments are channeled to companies that can best provide for individuals during times of need private programs that provide funding to the government during tough budgetary times a hospital cafeteria offers a fixed-price lunch consisting of a main course, a dessert, and a drink. if there are four main courses, three desserts, and six drinks to pick from, in how many ways can a customer select a meal consisting of one choice from each category?